« first day (1758 days earlier)      last day (3264 days later) » 

12:11 AM
Ahoy, @AlexWertheim. I finally got some good news today, I may have a job teaching for the next year.
 
12:21 AM
congrats!
 
Thanks, Mike! I apparently scared Alex away, oops.
What are you up to, over the summer @MikeMiller ?
 
12:43 AM
Sorry for the delay, I'm not paying much attention to chat right now, @pjs36. I'm doing a little travel to a summer school and a workshop and then when I get back from that I've got some teaching (and math, of course).
 
1:01 AM
@MikeMiller Gotcha, no worries at all. It sounds like you'll be staying busy, at least!
 
That's the goal!
 
I can't say I've ever "workshopped" though, so that's definitely cool
 
@BalarkaSen But then his answer suffices?
 
 
1 hour later…
2:07 AM
Is there such a thing as advanced trigonometry?
Maybe it's just Fourier analysis.
 
2:28 AM
sigh. There's another round of grading out of the way.
 
That's awesome, @pjs36! Glad to hear it :). Where at, if you don't mind my asking?
 
3:04 AM
Hello!!
If there is something called advanced trigonometry I would love to study it
 
you can try trigonometry on a sphere or in the hyperbolic plane instead of the Euclidean one
(it's a thing)
 
Hmm...
Hello@anon
 
hello
 
So what have you been doing these days (in maths ) @anon
About solenoids?
 
reskimming some hyperbolic stuff, beginning to look at "algebraic curves and riemann surfaces" by miranda but it's going slowly
 
3:10 AM
Hmm....nice
 
it only just occurred to me the other day that the cross-ratio [z,a;b,c] (or one definition of it at least - seems every source has its own definition) can be defined as the unique mobius transformation of z which sends (a,b,c) to (0,1,infinity).
 
3:35 AM
Hey @Kaj
 
Hey there
 
Forgot my name again :p@Kaj
 
haha, You're Sayan. Just kinda tired myself.
 
Heya @Kaj, what's cooking?
 
Just checking in and seeing if there's anything interesting on Main
 
3:38 AM
Just got a little schoolwork over myself, now going to hit Aluffi.
Sayan?
 
Nothing there ... The standard of questions are going very low
Yes..@Soham
 
Post some, then. :P
What are you doing?
 
Doing physics homework having burgers then i will think of some topology
 
Yeah, that's pretty standard. This one could use some more upvotes. It's so rare these days to see people having work in their OP: math.stackexchange.com/questions/1301995/…
 
I can't wait to have the field theory chapter done. :)
 
3:40 AM
Well I just proved something in topology
You done open sets @Soham
 
A little, and not too well.
 
Topology is open sets :P
2
 
Haha, I wouldn't know yet.
 
user147690
@Kaj Can you verify if what someone told me is true: Can the GCD of two polynomials be defined as the gcd of their degrees?
 
user147690
E.g. $p(x)=x^{15}+...$, $o(x)=x^{20},gcd(p,o)=5$
 
3:53 AM
No no. Take the gcd of two polynomials to be the monic polynomial of largest possible degree dividing both of the given polynomials.
 
user147690
How would you find the degree of that monic polynomial in general?
 
There is an analog to the Euclidean algorithm in polynomial rings over a field @AlexClark
 
gcd in the polynomial ring over which ring of scalars? Z? Q? R? C?
 
user147690
$\Bbb R[x]$
 
in any case, given any three natural numbers $a,b,c$ with $a,b\ge c$ there exist polynomials $f(x)$ and $g(x)$ such that $\deg f=a$ and $\deg g=b$ and $\deg(\gcd(f,g))=c$. Thus, literally the only thing the degrees of $f$ and $g$ can tell us is an upper bound for the degree of their gcd.
 
user147690
3:58 AM
Fair enough
 
user147690
I suppose I better go find this E.A analog
 
user147690
Actually I would guess it really just is E.A with long division of polynomials?
 
yes
 
Indeed
 
given any tuple $(f,g)$ one can replace either $f$ or $g$ by its remainder upon division by the other, then do it again and again until you get $(h,h)$. at least assuming your scalars are a field (this doesn't work in $\Bbb Z[x]$ for instance).
 
4:21 AM
@Soham you can think of the powerset iterated n times of all real numbers
What do you basically mean by the question @Soham
 
 
1 hour later…
5:29 AM
Anyone online right now?
When does a symmetric group have symmetric groups as subgroups?
For all $n > 3$?
 
Yes I am But i cant help you with symmetric groups sorry @Soham
 
if $n < m$ then there is a natural inclusion $S_n \subset S_m$
 
Wait, I think I didn't mean symmetric groups, exactly.
Why is there only one "multiplication table" for groups with $|G| \leq 3$?
 
because you can write it down?
alternatively, because there is only one group of order $p$, $p$ prime, up to isomorphism
 
It seems like if the order is $4$, there is a sort of "free" subsquare in the table that can permute without destroying the "Sudoku property".
Oh, yes.
So: the trivial group is unique, and $2$ and $3$ are prime.
 
user147690
5:37 AM
@MikeM Irreducible polynomials in $\Bbb R[x]$ are only those with complex roots and degree 1 polynomials right?
 
those quadratics with complex roots, yes
every polynomial in $\Bbb R[x]$ splits as a product of quadratic and linear factors
 
@Mike, so multiplication tables for groups of prime order have no subsquares that we can permute without losing "group-ness". Does that say anything about subgroups?
 
google lagrange's theorem
 
Hmm.
Thanks.
 
user147690
@MikeMiller Oh I see now, thanks
 
6:06 AM
@SohamChowdhury almost. again, I am asking you to stop obsessing about getting to algebraic topology as fast as possible and to learn topology thoroughly.
@MikeMiller did you see what I pinged you on bordism?
 
@BalarkaSen what does that entail?
that's my question.
 
what does what entail?
 
"learning topology thoroughly"
Munkres part 1?
 
well, do a bit of Simmons and then study Munkres!
 
at what point am I ready, O master?
:P
 
6:08 AM
ready to do what?
 
to move on, as it were.
anyhow, I think I'll finish groups today.
which is cool.
 
Does Aluffi talk about classification of groups?
 
finitely-generated abelian groups?
that's in the second groups chapter, after rings and modules.
 
that and a bit about semidirect products, sylow's theorem, etc.
 
(I believe)
@BalarkaSen I haven't read that chapter yet.
All three Sylows are in there.
 
6:12 AM
eh, I prefer having all those in a single chapter.
 
How can I regularize the sum from n = 1 to infinity of log(log(n))?
 
How far should I continue Simmons before starting with Munkres?
 
I know how to do so for log(n) (take the derivative of the zeta function at s=0) but I'm stuck on this one.
 
make sure you do all those. those are the real group theory.
 
Hm, yes.
 
6:13 AM
@SohamChowdhury I have no idea. I've forgotten what Simmons does. I guess you will have the motivations after reading a bit about metric spaces.
 
I just wanted a clear set of prereqs for starting an algebraic topology book. I'm not obsessed with getting there as quick as possible, and I won't leave any ideas half-baked in my head.
$\hskip -0.5in \text{this} \rightarrow$"do a bit of Simmons and study Munkres" is enough on the point-set side of things then?
 
@Soham If you are comfortable and know analysis then do till algebraic operators
 
Also, do you have any experience with Bredon, @BalarkaSen?
 
And my previous question?
 
6:16 AM
yes
 
@Balarka How much of algebra do i require for altop?
Anyways Hi@Balarka
 
meh, I am not going to suggest everyone how fast he can study everything and get to algebraic topology.
I have works to do
figure out yourself, I guess.
 
Fine
 
yeah, I figured he would get pissed.
Algebra is beautiful.
Do as much as you can.
@Rem. ^
@BalarkaSen what have you done today? I did a bit of "olympiad"-ish math today morning and then started going through all the problems in this chapter.
 
6:18 AM
Well homological algebra I guess Let me finish simmons first then I will go to DF and continue DF again
 
nothing much, yet. I have to think about a few problems I have been planning to think about for a few days.
 
After that I suppose I will decide which to do Algebraic Topology or axiomatic set theory
 
I should go. Thanks for the help, @Balarka.
@Rememberme Learn set theory and then answer my question on Main :)
 
I have learnt till that much
But i dont understand your question
@Soham
 
@Remember you don't need homological algebra to do algebraic topology. you won't get motivation for studying the former unless you do singular homology, on the contrary.
 
6:20 AM
Ahh.. Thats at the far end of DF
@Soham You have left many statements undefined. When you say that In this Bijection $\Bbb{N}\rightarrow \Bbb{R}$ You say that it loses countability I dont understand what you mean by "loses countability"
And what loses countability, I tried thinking about that statement of yours and got a rough idea but i feel you cant work with that rough idea
 
@robjohn we have that $\Omega$ is bounded
 
@Vrouvrou I know. So how does that help?
 
Hello@robjohn
 
@Rememberme hello
 
user147690
@Balarka do you think you could read over some proofs of mine later on? Just on finding all maximal ideals of $\Bbb R[x],\Bbb Z[x]$ and proving $\Bbb Z[i]$ is a PID
 
6:30 AM
@robjohn And $\mu$ is a finite radon measure
 
user147690
[the proofs aren't written yet, but I have my outlines mostly ready]
 
@robjohn so we have that $\mu(\Omega)<\infty $ no ?
 
@Vrouvrou yes, I said that.
8 hours ago, by robjohn
I guess it would have to be...
 
@robjohn what is the center of mass of $\mu$
please
 
In physics, the center of mass of a distribution of mass in space is the unique point where the weighted relative position of the distributed mass sums to zero. The distribution of mass is balanced around the center of mass and the average of the weighted position coordinates of the distributed mass defines its coordinates. Calculations in mechanics are often simplified when formulated with respect to the center of mass. In the case of a single rigid body, the center of mass is fixed in relation to the body, and if the body has uniform density, it will be located at the centroid. The center of...
 
6:36 AM
it is a constant
 
what is a constant?
 
center of mass ?
 
@AlexClark sure
 
@Vrouvrou $$\text{center of mass of }\Omega\text{ weighted by }\mu=\frac1{\mu(\Omega)}\int_\Omega x\,\mathrm{d}\mu$$
 
what were the maximal ideals of $\Bbb Z[x]$ you found?
 
6:40 AM
@robjohn is this is in $\overline{\Omega}$ ?
 
@Vrouvrou whatever domain you want
 
user147690
@BalarkaSen Hmmm haven't fully got this down yet, R[x] is ready to write up I imagine, and I have an idea how to write down the proof for $\Bbb Z[i]$ being a PID
 
@robjohn how ?
 
@Vrouvrou what do you mean "how"?
 
i dont understand whatever domain you want
we work on omega
 
6:56 AM
@Vrouvrou Fine... use $\Omega$ or $\overline{\Omega}$. Whatever domain you want to compute the center of mass of the measure.
 
 
1 hour later…
8:06 AM
I think this the best question with this tag:
http://math.stackexchange.com/questions/1302183/did-guinness-book-of-records-screw-this-up#1302183
 
8:53 AM
How can I prove that $A = \left( \begin{array}{ccc}
1 & 1 \\
0 & 1 \end{array} \right)$ has infinite order in $\text{GL}_2\mathbb{R}$?
 
Hey everyone. Can someone please answer this question for me.
0
Q: Help understanding the range and kernel of a linear transformation

nTuplyI'm having some trouble understanding the Range and Kernel of a linear transformation. The definition goes as follows: Let $T:V \longrightarrow W$ be a linear transformation. Define the sets $ker(T) = \{v\in V : T(v) =0\}$, $R(T) =\{w\in W : w=T(v)\; for\; some \; v\in V\}$. $ker(T)$ is the k...

 
I missed the brackets, sorry.
I tried using determinants, but that doesn't help in this case because $A$ has a det of 1.
 
@SohamChowdhury what does it mean to have infinite order?
 
No power is equal to the identity.
$I_2$, in this case.
 
right. so can you find an explicit formula for $A^n$?
using induction, maybe?
 
9:06 AM
Wait.
 
and then show that $A^n \neq I$
 
Is this the Fibonacci matrix?
 
who cares about big names
2
 
Shiiiit.
 
just find out an explicit formula for $A^n$. compute $A^2, A^3, A^4$ first.
and then induct something out.
 
9:09 AM
Oh. $\left(\begin{array}{ccc} 1 & n \\ 0 & 1\end{array}\right)$
 
mhm
 
So, not the Fibonacci matrix. (Which is cool btw, you can use it to prove Cassini's identity, I found.)
Thanks
For odd-order $g \in G$, does $g^2$ also have odd order?
I guess so.
 
prove it.
 
I'm doing that rn
 
@SohamChowdhury didn't even know that thing was called fibonacci matrix
 
9:15 AM
I'm not sure it is called that, but what else can you call it if not that?
Bonaccir chheler matrix?
Okay, I think I got a proof.
 
why would you need to call it anything in the first place? :P
 
@BalarkaSen Right on the money :p
 
So let $0\leq r < |g|.$
$g^{2|g| - r} = g^{2|g|}\cdot g^{-r} = g^{-r} = (g^{-1})^r = e.$
This implies that $|g^{-1}| < |g|$.
This is a contradiction, except if $r = 0$. This implies $|g^2| = |g|$.
Something's wrong.
@AlexClark, online?
 
user147690
@SohamChowdhury Yes but depressed due to non-math things, so I am taking a break
 
user147690
But I'll have a coffee and get back to it in 10 I suppose
 
9:25 AM
Would it be too much to ask for you to look at the "proof"?
Okay, that's awesome.
That does not imply $|g^2| = |g|$.
 
9:47 AM
I seem to have lost my mind. I just made up a multiplication table for a group of order 3 with elements of order 2.
Isn't this impossible?
 
Well I am happy today!!
I dont know how
 
Good.
Oh, never mind.
It was wrong.
 
user147690
@Rememberme Why are you happy today?
 
I have got a 10 in my school results@AlexC
 
@Rememberme Awesome.
@AlexClark Help me, I seem to be going mad.
 
user147690
9:53 AM
@Rememberme 10 out of what?
 
user147690
@SohamChowdhury I can try, but maybe madness is contagious!
 
user147690
Sure you don't mean \hrule
 
user147690
Nvm they are both things
 
So, essentially this is a group multiplication table.
$\begin{array} - & e & a & b \\ e & e & a & b \\ a & b & e & a \\ b & a & b & e\end{array}$
All the elements seem to have order 2.
What's wrong?
 
user147690
9:56 AM
So it's non commutative
 
@AlexC?
But a group of order 3 can't have elements of order 2.
 
user147690
What is your identity?
 
Lagrange's theorem.
e.
 
user147690
Looks like you don't have one
 
user147690
Check your identity again
 
9:57 AM
Oho.
Right.
Told you I was going mad.
I need a coffee too, I guess.
 
user147690
Haha that's fine
 
"If $|g|$ is odd, what can you say about $|g^2|$?"
I'll work on this now.
 
user147690
What is $g$?
 
An element.
 
user147690
Enjoying group theory? I think it is one of the better topics. Are you working from Aluffi for this?
 
10:01 AM
Yes, I am. Yes, I am.
But brief periods of incompetence are painful and disturbing. You'd know, from the other day.
Oh, it's an easy proof.
$$|g^2| = \frac{|g|}{\gcd(2, |g|)} \overset{|g| \;\text{is odd}}{=} \frac{|g|}{1} = |g| $$.
@BalarkaSen
 
user147690
10:23 AM
Wow so empty here, 6 members
 
Now it's 7.
 
10:36 AM
Is that correct?
Paging @BalarkaSen
 
yep
 
Mmm.
"If g and h commute and the gcd of their orders is 1, the order of their product is the product of their orders."
I got till $|gh| = k|g||h|$.
 
user147690
So the only maximal ideals of $\Bbb R[x]$ are of the form $(x-a)$ and $(x^2-2ax+(a^2+b^2))$
 
Fact : If $g, h$ commutes then $(gh)^n = h^ng^n$
Prove this.
@AlexClark Yes.
 
I can't do $(gh)^n = g^nh^n$, obviously.
 
user147690
10:39 AM
@BalarkaSen Feels kinda anticlimatic
 
Wait, no, what about $x^4 + 1$?
Irreducible polynomials over $\Bbb R$ are surely not just quadratics and linear things.
 
Can I use that, @Balarka?
 
user147690
@BalarkaSen Over $\Bbb R[x]$ the maximals seem to be
 
@SohamChowdhury Well, why not?
 
Then $(gh)^n = (hg)^n = h^ng^n$.
 
10:41 AM
$(x^4 + 1)$ is maximal, but yet not what you say it is.
Yes, @Soham. What's the contradiction?
 
user147690
@BalarkaSen $x^4+1=-(x^2+\sqrt{2}x-1)(x^2+\sqrt{2}x+1)$ so it is generated by $(x^2+\sqrt{2}x+1)$
 
No, I thought I would have to prove that separately.
 
Yikes, yes, need coffee.
@SohamChowdhury You're not making sense. I said that $g$ and $h$ commutes. So $(gh)^n = g^nh^n = h^ng^n$
 
And that's easy too. $(gh)^n = ghghgh\cdots = gggg\cdots hhhh\cdots = g^nh^n$ if gh commute.
No, I thought I couldn't assume $(gh)^n = g^nh^n$. That's why I said that.
 
Right. So if $n$ is the order of $gh$, then $n$ is the order of $g$ and $h$ too.
Done.
@SohamChowdhury What's to assume? It's a provable fact.
 
10:44 AM
How is that, now?
@BalarkaSen Yes, I thought I had to supply the proof.
 
I don't get whatever you're trying to say.
 
No, it's like what happened yesterday with norms.
I was sort of assuming the answer and saying something like "we know that x is true, so x."
Anyway, miscommunication can't be helped sometimes.
@BalarkaSen "How is that?" was in reference to the fact that I didn't understand this message. (linked)
How does that follow from the previous fact?
Oh, I see.
 
no, that was a typo.
OK, so what was the thing we wanted to prove again?
 
user147690
Usually when my conversations get so muddled I start again lol
 
@BalarkaSen $(gh)^n = g^nh^n$ for commuting $g,h$.
 
10:48 AM
No, no that was the hint I have you.
 
$|gh| = |g||h|$.
 
What was your problem?
Right.
So assume $|g| = n$, $|h| = m$
We know that $(gh)^k = g^kh^k$.
 
Should I continue?
 
Sure.
 
I'm not really sure what to do. I can show that $(gh)^{mn} = e$: $(gh)^n = g^nh^n = h^n \implies (gh)^{mn} = (h^n)^m = e$.
But how is it minimal?
 
10:52 AM
Start by assuming it's not.
 
The division algorithm trick?
Let $0\leq k<mn$.
 
We already know that $(gh)^{mn} = 1$. If $(gh)^k$ for some $k$ smaller than $mn$, then $k | mn$ must hold.
Recall that you have some restriction on $m, n$
Use that.
 
Um, then, wlog, $k | m$? Because $\gcd(m,n) = 1$?
I have no clue what to do from there.
 
Think, think.
 
Am I moving in the right direction?
 
11:00 AM
Yes, you are.
 
I had copied this text from a while back. Don't laugh.
$(gh)^{mn-k} = e \implies gh^{mn}\cdot((gh^{-1})^k) = e \implies |gh| = k$. I can replace ${mn}$ by $k$ and find $k_1$, and so on. Infinite descent?
I'm thinking.
btw, we haven't even proved Lagrange's theorem yet in the text.
 
You don't need Lagrange to do this.
 
Not Lagrange, but isn't $|g|$ divides $|G|$ a corollary or something?
We haven't proved it yet, anyhow.
 
Where have I used that?
 
Oh, never mind.
$k|m$. What next?
 
11:09 AM
Anyway, you're thinking too hard. $k | m$, so $e = (gh)^k = g^k h^k$. This means $gh = 1$. However, that contradicts $(|g|, |h|) = 1$
 
@AlexClark 10 ... This is something we call cgpa cumalative grade point nonsense I don't know the last word so that is out of 10 which we get when we finish a whole year which is out of 10 in that I got a 10
 
user147690
@Rememberme average lol(I mean it is cumulative grade point average). Good work!
 
Yes some shit
 
@AlexClark Whoa, you risked being misunderstood there lol
 
user147690
@SohamChowdhury Yes I realised when I finished reading whoops!
 
11:10 AM
Right
 
user147690
Sorry if I offended you for a second there Sayan haha
 
Equivalently, note that $g^k \neq 1$ as $k < n$, and $h^k \neq 1$ as $(|h|, |g|) = 1$. So $g^kh^k = 1$ is absurd.
 
This might seem idiotic but o will call two open spheres the same of the radius of both of them are same
Right
 
@BalarkaSen Because $gh = 1$ implies both have equal orders?
 
Or nonsense
 
11:12 AM
@SohamChowdhury You really should think about a problem (for hours, yes) before asking questions in here.
yes, @Soham
 
user147690
I have no idea what you just said @Rem
 
@BalarkaSen I know, I know :(
 
user147690
"This might seem idiotic but o will call two open spheres the same of the radius of both of them are same"
 
"o" $\mapsto$ "we"
 
I am on my phone so sorry for bad grammar
 
11:13 AM
Hammer?
 
user147690
" Of" $\mapsto$ "if"?
 
yes, your hrammer has always been mucked up
 
user147690
What do you mean by open spheres? Open balls?
 
user147690
Are they located at the exact same point?
 
Nope
 
user147690
11:14 AM
They are at different points, then by what means do you mean call them the same?
 
Well I thought this because we call two circles equal if they are of the same radius
 
@BalarkaSen What if $g$ and $h$ are inverses of each other?
 
So I thought that might imply to open spheres
 
user147690
@Rememberme I don't even know what it means to name them haha
 
@SohamChowdhury Can't be : we assumed $(|g|, |h|) = 1$. That's the whole point.
 
user147690
11:16 AM
In my mind they should only be 'equal' if they are the same, e.g. same centre and same radius
 
Well I will ask you again when I am on my pc
 
@BalarkaSen What if they're both = 1?
We assume they're different, then?
 
user147690
@Rememberme Ok
 
Typing on the phone is literally difficult
 
user147690
@Rememberme I know haha, always frustrates me
 
11:17 AM
Everything gets muddled up when you assume they are identity :P
 
Again wrong grammar
 
Treat that case separately.
 
@BalarkaSen Ah, now I feel like I understand. Thanks a ton, man.
 
Okay well my battery is going to go dead talk later
 
Just a suggestion : it'd be better if you do a bit of elementary number theory before doing algebra deeply.
Not a lot : pick up Niven-Zuckerman-Montgomery and do exercises from the first few chapters.
 
11:19 AM
Yes @SohamChowdhury balarka is right over there
 
@BalarkaSen I have Burton.
Will that do?
 
doesn't have good exercises, I think
 
Well exercises are very easy in it
 
Also when you go ahead in it it feels like burton rushes
 
11:25 AM
I really need to brush up on elementary NT anyhow, so that's actually a very good thing you reminded me of.
I have a lot of trouble following that kind of proof, even though I'm familiar with the material to some extent.
 
11:50 AM
Last exercise!
Let $G$ be an abelian group, and let $g$ be an element of max finite order. Show that for any other $h$ with finite order, $|h|$ divides $g$.
@BalarkaSen, here's my proof.
The hint is this: "calculate $|g^{p^m}h^s|$".
$$|g^{p^m}h^s| = |g^{p^m}||h^s| \\
= \frac{|g|}{\gcd(|g|,p^m)} \times \frac{|h|}{\gcd(|h|,s)} \\
= \frac{|g||h|}{p^ms} = p^nr$$.
$r > s \implies p^nr > p^mr$, so this element has order greater than $g$, contradiction.
We are considering $g=p^mr, h=p^ms$ for prime p and relatively prime $r,s$ and $m<n$.
 
@SohamChowdhury First line is not right
the order of a product need not be the product of the orders
 
I know.
 
But that is what the first line says
 
But the gcd of the orders is 1.
I haven't justified it yet.
$|g^{p^m}| = r, |h^s| = s$, so $\gcd(|g^{p^m}|, |h^s|) = 1$.
 

« first day (1758 days earlier)      last day (3264 days later) »